Đến nội dung

an1712

an1712

Đăng ký: 18-04-2015
Offline Đăng nhập: 23-08-2023 - 09:44
****-

#569934 $\sum \frac{a^{2}}{a^{2}+ab...

Gửi bởi an1712 trong 04-07-2015 - 20:22

Cho 3 so $a,b,c>0$ 

Chung minh rang $\sum \frac{a^{2}}{a^{2}+ab+b^{2}}\geq 1$

http://diendantoanho...ác-nước/page-23

đã có tại đây




#569905 $\frac{1+a^{2}}{1+b+c^{2}}+...

Gửi bởi an1712 trong 04-07-2015 - 17:03

Cho $a,b,c$ là các số thực thỏa mãn : $a,b,c>-1$ . Chứng minh rằng : 

$\frac{1+a^{2}}{1+b+c^{2}}+\frac{1+b^{2}}{1+c+a^{2}}+\frac{1+c^{2}}{1+a+b^{2}}\geq 2$

 

Spoiler

ta có:

$\frac{b^2+1}{2}\geq b \Leftrightarrow \frac{a^2+1}{1+b+c^2}\geq \frac{a^2+1}{\frac{3}{2}+\frac{b^2}{2}+c^2}$

áp dụng CBS

$\sum \frac{a^2+1}{\frac{3}{2}+\frac{b^2}{2}+c^2}\geq \frac{(a^2+b^2+c^2+3)^2}{3(\sum a^2)+\frac{3}{2}(\sum a^2b^2)+\frac{9}{2}}\geq 2$

$\Leftrightarrow (a^2+b^2+c^2)^2\geq 3(\sum a^2b^2)$ hiển nhiên




#569718 Topic tổng hợp một số bất đẳng thức trong kì thi MO các nước

Gửi bởi an1712 trong 03-07-2015 - 20:57

Bài 168(Hungary MO): Cho a,b,c>0. CMR nếu a+b+c= 2(ab+ca+ab). Tìm Min biểu thức sau:

P=$\sum \frac{a^3+b^2}{a^2+b^2}$

  ta có;$a+b+c=2(ab+bc+ac)\leq \frac{2}{3}(a+b+c)^2\Leftrightarrow \frac{3}{2}\leq (a+b+c)$

  giả sử :

      $a\geq c\geq b$  $\Leftrightarrow$ $a\geq \frac{1}{2}$

$a+b-ab=2c(a+b-\frac{1}{2})+ab> 0$

$\sum \frac{a^3+b^2}{a^2+b^2}=\sum \frac{a^3-a^2}{a^2+b^2}+3$

xét hiệu:

$\frac{a^3-a^2}{a^2+b^2}+\frac{b^3-b^2}{b^2+c^2}\geq \frac{a^3-a^2}{a^2+c^2}+\frac{b-1}{2}$

 

$\Leftrightarrow (a^3-a^2)(\frac{c^2-b^2}{(a^2+b^2)(a^2+c^2)})+(b^3-b^2)(\frac{b^2-c^2}{2b^2(b^2+c^2)})\geq 0$

 

$\Leftrightarrow \frac{a^3-a^2}{(a^2+b^2)(a^2+c^2)}\geq \frac{b^3-b^2}{2b^2(b^2+c^2)}=\frac{b-1}{2(b^2+c^2)}$

 

$\Leftrightarrow 2(a^3b^2+a^3c^2)+a^4+a^2b^2+a^2c^2+b^2c^2\geq a^4b+b^3a^2+a^2c^2b+b^3c^2+2(a^2b^2+a^2c^2)$

 

$\Leftrightarrow (a-b)(a^2b^2+2a^2c^2+b^2c^2+(a+b-ab)(a^2-c^2))\geq 0$

  

$P\geq \frac{a^3-a^2}{a^2+c^2}+\frac{c^3-c^2}{a^2+c^2}+\frac{b-1}{2}+3\geq \frac{a^3+c^3}{a^2+c^2}+\frac{b}{2}+\frac{3}{2}$

lại có:

$a^3+c^3\geq ac(a^2+c^2)\Leftrightarrow 2(a^3+c^3)\geq (a+c)(a^2+c^2)\Leftrightarrow a^3+c^3\geq \frac{(a+c)(a^2+c^2)}{2}$

 

$\Rightarrow P\geq \sum \frac{a}{2}+\frac{3}{2}\geq \frac{9}{4}$




#569106 Topic tổng hợp một số bất đẳng thức trong kì thi MO các nước

Gửi bởi an1712 trong 30-06-2015 - 15:56

Bài 159(Balkan Shortlist): Cho a,b,c >0. CMR:

$\sum \frac{a^2}{a^2+ab+b^2}\geq 1$

đặt $\frac{b}{a}=x $ $\frac{c}{b}=y $ $\frac{a}{c}=z $ $\Leftrightarrow  xyz=1$

bdt$\Leftrightarrow \sum \frac{1}{1+x+x^2}\geq 1$

đặt$x=\frac{y_{1}z_{1}}{x_{1}^2}$.........

bđt $\Leftrightarrow \sum \frac{x_{1}^4}{x_{1}^4+x_{1}^2y_{1}z_{1}+y_{1}^2z_{1}^2}\geq 1$

$\Leftrightarrow \sum x_{1}^2y_{1}z_{1}\leq \sum x_{1}^2y_{1}^2$ luôn đúng




#568308 Topic tổng hợp một số bất đẳng thức trong kì thi MO các nước

Gửi bởi an1712 trong 26-06-2015 - 15:33

Giả sử $a=\text{max}\{a,b,c\}$ thì $f(a,b,c)-f(a,\sqrt{bc},\sqrt{bc})=(\sqrt{b}-\sqrt{c})^2\left[a^3(b+c+\sqrt{bc})^2-6\right]\geqslant (\sqrt{b}-\sqrt{c})^2(9a^2-6)\geqslant 0$

Do đó ta chỉ cần chứng minh bất đẳng thức khi $a\geqslant b=c=t$. Thay trực tiếp $a$ theo $t$ và khảo sát hàm số.

a xin góp 1 bài dồn biến, ko nghĩ bài này mạnh vậy,dùng shur bị ngược dấu.

ko mất tính tổng quát :$c\geq 1\Leftrightarrow ab\leq 1$

xét hàm : $\sqrt{\frac{b}{a}}\geq t\geq 1$

$f(t)=\frac{1}{a^3t^3}+\frac{t^3}{b^3}+\frac{1}{c^3}-6(at+\frac{b}{t}+c)$

=> $f'(t)=\frac{3(at^2-b)(a^2t^4+abt^2+b^2-2t^2a^3b^3)}{a^3b^3t^4}\geq \frac{3(at^2-b)(3abt^2-2a^3b^3t^2)}{a^3b^3t^4}$

mà $ab\geq a^3b^3$ $(vì ab\leq 1)$ và $at^2-b\leq 0$

$\Leftrightarrow f'(t)\leq 0$ (nghịch biến)

$\Rightarrow f(1)\geq f(\sqrt{\frac{b}{a}})$

$\Leftrightarrow f(a,b,c)\geq f(\sqrt{ab},\sqrt{ab},c)$

đưa hàm t rồi tìm gtnn




#567866 Topic tổng hợp một số bất đẳng thức trong kì thi MO các nước

Gửi bởi an1712 trong 24-06-2015 - 16:30

Bài 150(Iran TST): Cho a,b,c>0: abc=1. CMR:
$\frac{a}{c+a(a^3+b^3)}+\frac{b}{a+b(b^3+c^3)}+\frac{c}{b+c(c^3+a^3)}\leq 1$

có:$(c+a(a^3+b^3))(c+\frac{bc}{a}+c)=(c+\frac{a^3}{bc}+\frac{b^2}{c})(2c+\frac{bc}{a})\geq (a+b+c)^2$

bdt $\Leftrightarrow \frac{3(\sum ab)}{(a+b+c)^2}\leq 1$ (luôn đúng)




#567639 Topic tổng hợp một số bất đẳng thức trong kì thi MO các nước

Gửi bởi an1712 trong 23-06-2015 - 14:58

 

$(a^2+b^2c^2+c^2)(b^2+1+a^2)\geq (ab+bc+ac)^2=9$

 

cái đó hình như bị nhầm rồi biểu thức là $a^2+b^2(c^2+1)=a^2+b^2c^2+b^2$




#567609 Topic tổng hợp một số bất đẳng thức trong kì thi MO các nước

Gửi bởi an1712 trong 23-06-2015 - 11:15

Câu 13 :(Balkan MO , 2014) Cho $x,y,z>0$ thỏa : $xy+yz+zx=3xyz$ . Chứng minh rằng : 

$x^{2}y+y^{2}z+z^{2}x\geq 2(x+y+z)-3$

 

P/s : Mong các bạn tham gia sôi nổi nhé ! :biggrin:

áp dụng holder:

$(x^2y+y^2z+z^2x)(\sum \frac{1}{x})\geq (x+y+z)^2$ 

$\sum x^2y\geq \frac{xyz(x+y+z)^2}{xy+yz+xz}\doteq \frac{(x+y+z)^2}{3}$

áp dụng cố si:

$\frac{(x+y+z)^2}{3}+3\geq 2(x+y+z) (đpcm)$




#567354 Topic tổng hợp một số bất đẳng thức trong kì thi MO các nước

Gửi bởi an1712 trong 21-06-2015 - 22:10

Bài 134:(Baltic Way Shortlist) :Cho $a,b,c>0: a+b+c=1$. CMR:

$\frac{a^3+bc^2+ca^2}{b(c+2a)}+\frac{b^3+ca^2+ab^2}{c(a+2b)}+\frac{c^3+ab^2+bc^2}{a(b+2c)}\geq \frac{216abc+1}{9}$

ta có :

$\sqrt[3]{3b(2c+a)(2a+c)}\leq a+b+c$

$\Leftrightarrow \sum \frac{a^3+bc^2+ca^2}{b(c+2a)}\geq \sum \frac{3(2c+a)(a^3+bc^2+ca^2)}{(a+b+c)^3}$

$bđt \Leftrightarrow 3((a^2+b^2+c^2)^2+5(ab^3+bc^3+ca^3)+abc)\geq \frac{216abc+1}{9}$

áp dụng holder:

$(ab^3+bc^3+ca^3)(\sum \frac{1}{a})(1+1+1)\geq (a+b+c)^3=1$ $\Leftrightarrow 5(\sum ab^3)\geq \frac{5abc}{ab+bc+ac}$

đặt : $a+b+c=p=1$ $ab+ac+bc=q\leq \frac{1}{3}$ $abc=r$

$bdt\Leftrightarrow (p^2-2q)^2+\frac{5r}{3q}-7r-\frac{1}{27}\geq 0\Leftrightarrow (1-2q)^2+\frac{5r}{3q}-7r-\frac{1}{27}$

xét hàm :

TH1

$\frac{5}{21}\geq q$ hàm f'(r) đồng biến

theo shur :$r\geq \frac{4q-1}{9}$

$\Rightarrow f(r)\geq f(\frac{4q-1}{9})\geq 0$ (luôn đúng)

TH2

$\frac{5}{21}\leq q\leq \frac{1}{3}$ (nghịch biến)

có:$\prod (a-b)^2\geq 0\Leftrightarrow \frac{(1+2\sqrt{1-3q})(1-\sqrt{1-3q})^2}{27}\geq r$

$f(r)\geq f(\frac{(1+2\sqrt{1-3q})(1-\sqrt{1-3q})^2}{27})\geq 0$(luôn đúng)

=> đpcm




#567232 Tìm GTNN của: $P=a+b+c$

Gửi bởi an1712 trong 21-06-2015 - 08:34

Bạn có thể giải thích vể cách làm của bạn được không ?

cách của bạn ấy là đổi biến để dễ áp dụng AM_GM




#567105 Topic tổng hợp một số bất đẳng thức trong kì thi MO các nước

Gửi bởi an1712 trong 20-06-2015 - 15:42

Bài 124:(IMO Shortlist) : Cho a,b,c là số đo 3 cạnh một tam giác. CMR:

 

$\frac{a^3}{b^2(c+a-b)}+\frac{b^3}{c^2(a+b-c)}+\frac{c^3}{a^2(b+c-a)}\geq 3$

áp dụng holder:

$(\sum \frac{a^3}{b^2(c+a-b)})(\sum bc+ab-b^2)(a+b+c)\geq (a+b+c)^3$

$bđt\Leftrightarrow \frac{(a+b+c)^2}{2(ab+bc+ac)-(a^2+b^2+c^2)}\geq 3$

$\sum a^2\geq \sum ab$ hiển nhiên




#567014 Topic tổng hợp một số bất đẳng thức trong kì thi MO các nước

Gửi bởi an1712 trong 20-06-2015 - 06:59

Câu 6: Cho $a,b,c$ là các số thực dương. CMR:

$a^3b^6+b^3c^6+c^3a^6+3a^3b^3c^3\geq abc(a^3b^3+b^3c^3+c^3a^3)+a^2b^2c^2(a^3+b^3+c^3)$ (BMO 2015)

chia 2 vế cho $a^3b^3c^3$

bđt$\Leftrightarrow \sum \frac{b^3}{c^3}+3\geq \sum \frac{ab}{c^2}+\sum \frac{a^2}{bc}$

đặt $\frac{b}{c}=x$ $\frac{c}{a}=y$ $\frac{a}{b}=z$ 

$\Leftrightarrow x^3 +y^3 +z^3+3\geq \sum \frac{x+y}{z}$

mà xyz=1$\Leftrightarrow x^3 +y^3 +z^3+3xyz\geq xy(x+y)+yz(y+z)+xz(x+z)$ (luôn đúng theo shur)




#567012 Tìm min $P=\frac{\sqrt{3(2x^2+2x+1)}}...

Gửi bởi an1712 trong 20-06-2015 - 06:19

Câu này được đăng lên đây rất lâu rồi (không nhớ rõ)

$\frac{1}{\sqrt{2x^2+(3-\sqrt{3})x+3}}+\frac{1}{\sqrt{2x^2+(3+\sqrt{3})x+3}}\geq \frac{4}{\sqrt{2(4x^2+6x+6)}}$

 

bạn dùng bđt gì vậy, chỉ mk vs




#566963 Tìm min $P=\frac{\sqrt{3(2x^2+2x+1)}}...

Gửi bởi an1712 trong 19-06-2015 - 21:02

Xét số thực x. Tìm min của

$P=\frac{\sqrt{3(2x^2+2x+1)}}{3}+\frac{1}{\sqrt{2x^2+(3-\sqrt{3})x+3}}+\frac{1}{\sqrt{2x^2+(3+\sqrt{3})x+3}}$

mk đã lm thử bài này hồi trc, nếu giải bằng xét hàm số thì rất rắc rối nhưng lời giải là bằng hình học rất ko tự nhiên




#566893 Topic tổng hợp một số bất đẳng thức trong kì thi MO các nước

Gửi bởi an1712 trong 19-06-2015 - 16:56

 

Bài 116 (CĐTMO 2005) : Chứng minh rằng
                                                                 $\frac{a^{3}}{(b+c)^{3}}+\frac{b^{3}}{(c+a)^{3}}+\frac{c^{3}}{(a+b)^{3}}$ $\geq \frac{3}{8}$

                                        trong đó $a,b,c$ là các số dương.

Spoiler


 

ta có $8(x-\frac{1}{2})^2(x+1)\geq 0\Leftrightarrow 8x^3\geq 6x-2$

$\Rightarrow \sum \frac{8a^3}{(b+c)^3}\geq 6(\sum \frac{a}{b+c})-6\geq 3$ theo nesbit

$\Rightarrow P\geq \frac{3}{8}$